Sum of two co-prime integers












3












$begingroup$


I need some help in a proof:
Prove that for any integer $n>6$ can be written as a sum of two co-prime integers $a,b$ s.t. $gcd(a,b)=1$.



I tried to go around with "Dirichlet's theorem on arithmetic progressions" but didn't had any luck to come to an actual proof.
I mainly used arithmetic progression of $4$, $(4n,4n+1,4n+2,4n+3)$, but got not much, only to the extent of specific examples and even than sometimes $a,b$ weren't always co-prime (and $n$ was also playing a role so it wasn't $a+b$ it was $an+b$).



I would appriciate it a lot if someone could give a hand here.










share|cite|improve this question











$endgroup$








  • 4




    $begingroup$
    What's wrong with $n=1+(n-1)$? Perhaps you meant to require $a,b>1$ but you should specify that.
    $endgroup$
    – lulu
    Dec 15 '18 at 16:03








  • 1




    $begingroup$
    Dirichlet’s theorem is useless in that kind of situation; the problem is much, much simpler. Aside from the obvious case $n=(n-1)+1$, you can try to find general patterns when $n=4m$, $n=2m+1$, $n=4m+2$.
    $endgroup$
    – Mindlack
    Dec 15 '18 at 16:05






  • 1




    $begingroup$
    True i should have specified that a,b>1, sorry, my bad.
    $endgroup$
    – Daniel Gimpelman
    Dec 15 '18 at 16:07










  • $begingroup$
    Daniel, you can edit your post. Also, have a look at this introduction to posting mathematical notation.
    $endgroup$
    – hardmath
    Dec 15 '18 at 16:12










  • $begingroup$
    Mindlack thanks, but I don't really understand how to go from here. How do I represent a and b even in a specific case? $7=2*3+1$ where $a=2$ and $b=1$? $8=4*2$ where $a=4$ and $b=0$? (And than b isn't co-prime). $10=4*2+2$ where $a=4$ and $b=2$ (a,b aren't co-prime). Also there would always be the m, which i'm not sure if the m could be greater than 1.
    $endgroup$
    – Daniel Gimpelman
    Dec 15 '18 at 16:25


















3












$begingroup$


I need some help in a proof:
Prove that for any integer $n>6$ can be written as a sum of two co-prime integers $a,b$ s.t. $gcd(a,b)=1$.



I tried to go around with "Dirichlet's theorem on arithmetic progressions" but didn't had any luck to come to an actual proof.
I mainly used arithmetic progression of $4$, $(4n,4n+1,4n+2,4n+3)$, but got not much, only to the extent of specific examples and even than sometimes $a,b$ weren't always co-prime (and $n$ was also playing a role so it wasn't $a+b$ it was $an+b$).



I would appriciate it a lot if someone could give a hand here.










share|cite|improve this question











$endgroup$








  • 4




    $begingroup$
    What's wrong with $n=1+(n-1)$? Perhaps you meant to require $a,b>1$ but you should specify that.
    $endgroup$
    – lulu
    Dec 15 '18 at 16:03








  • 1




    $begingroup$
    Dirichlet’s theorem is useless in that kind of situation; the problem is much, much simpler. Aside from the obvious case $n=(n-1)+1$, you can try to find general patterns when $n=4m$, $n=2m+1$, $n=4m+2$.
    $endgroup$
    – Mindlack
    Dec 15 '18 at 16:05






  • 1




    $begingroup$
    True i should have specified that a,b>1, sorry, my bad.
    $endgroup$
    – Daniel Gimpelman
    Dec 15 '18 at 16:07










  • $begingroup$
    Daniel, you can edit your post. Also, have a look at this introduction to posting mathematical notation.
    $endgroup$
    – hardmath
    Dec 15 '18 at 16:12










  • $begingroup$
    Mindlack thanks, but I don't really understand how to go from here. How do I represent a and b even in a specific case? $7=2*3+1$ where $a=2$ and $b=1$? $8=4*2$ where $a=4$ and $b=0$? (And than b isn't co-prime). $10=4*2+2$ where $a=4$ and $b=2$ (a,b aren't co-prime). Also there would always be the m, which i'm not sure if the m could be greater than 1.
    $endgroup$
    – Daniel Gimpelman
    Dec 15 '18 at 16:25
















3












3








3





$begingroup$


I need some help in a proof:
Prove that for any integer $n>6$ can be written as a sum of two co-prime integers $a,b$ s.t. $gcd(a,b)=1$.



I tried to go around with "Dirichlet's theorem on arithmetic progressions" but didn't had any luck to come to an actual proof.
I mainly used arithmetic progression of $4$, $(4n,4n+1,4n+2,4n+3)$, but got not much, only to the extent of specific examples and even than sometimes $a,b$ weren't always co-prime (and $n$ was also playing a role so it wasn't $a+b$ it was $an+b$).



I would appriciate it a lot if someone could give a hand here.










share|cite|improve this question











$endgroup$




I need some help in a proof:
Prove that for any integer $n>6$ can be written as a sum of two co-prime integers $a,b$ s.t. $gcd(a,b)=1$.



I tried to go around with "Dirichlet's theorem on arithmetic progressions" but didn't had any luck to come to an actual proof.
I mainly used arithmetic progression of $4$, $(4n,4n+1,4n+2,4n+3)$, but got not much, only to the extent of specific examples and even than sometimes $a,b$ weren't always co-prime (and $n$ was also playing a role so it wasn't $a+b$ it was $an+b$).



I would appriciate it a lot if someone could give a hand here.







number-theory elementary-number-theory






share|cite|improve this question















share|cite|improve this question













share|cite|improve this question




share|cite|improve this question








edited Dec 15 '18 at 17:17









Avi Steiner

2,708927




2,708927










asked Dec 15 '18 at 16:01









Daniel GimpelmanDaniel Gimpelman

184




184








  • 4




    $begingroup$
    What's wrong with $n=1+(n-1)$? Perhaps you meant to require $a,b>1$ but you should specify that.
    $endgroup$
    – lulu
    Dec 15 '18 at 16:03








  • 1




    $begingroup$
    Dirichlet’s theorem is useless in that kind of situation; the problem is much, much simpler. Aside from the obvious case $n=(n-1)+1$, you can try to find general patterns when $n=4m$, $n=2m+1$, $n=4m+2$.
    $endgroup$
    – Mindlack
    Dec 15 '18 at 16:05






  • 1




    $begingroup$
    True i should have specified that a,b>1, sorry, my bad.
    $endgroup$
    – Daniel Gimpelman
    Dec 15 '18 at 16:07










  • $begingroup$
    Daniel, you can edit your post. Also, have a look at this introduction to posting mathematical notation.
    $endgroup$
    – hardmath
    Dec 15 '18 at 16:12










  • $begingroup$
    Mindlack thanks, but I don't really understand how to go from here. How do I represent a and b even in a specific case? $7=2*3+1$ where $a=2$ and $b=1$? $8=4*2$ where $a=4$ and $b=0$? (And than b isn't co-prime). $10=4*2+2$ where $a=4$ and $b=2$ (a,b aren't co-prime). Also there would always be the m, which i'm not sure if the m could be greater than 1.
    $endgroup$
    – Daniel Gimpelman
    Dec 15 '18 at 16:25
















  • 4




    $begingroup$
    What's wrong with $n=1+(n-1)$? Perhaps you meant to require $a,b>1$ but you should specify that.
    $endgroup$
    – lulu
    Dec 15 '18 at 16:03








  • 1




    $begingroup$
    Dirichlet’s theorem is useless in that kind of situation; the problem is much, much simpler. Aside from the obvious case $n=(n-1)+1$, you can try to find general patterns when $n=4m$, $n=2m+1$, $n=4m+2$.
    $endgroup$
    – Mindlack
    Dec 15 '18 at 16:05






  • 1




    $begingroup$
    True i should have specified that a,b>1, sorry, my bad.
    $endgroup$
    – Daniel Gimpelman
    Dec 15 '18 at 16:07










  • $begingroup$
    Daniel, you can edit your post. Also, have a look at this introduction to posting mathematical notation.
    $endgroup$
    – hardmath
    Dec 15 '18 at 16:12










  • $begingroup$
    Mindlack thanks, but I don't really understand how to go from here. How do I represent a and b even in a specific case? $7=2*3+1$ where $a=2$ and $b=1$? $8=4*2$ where $a=4$ and $b=0$? (And than b isn't co-prime). $10=4*2+2$ where $a=4$ and $b=2$ (a,b aren't co-prime). Also there would always be the m, which i'm not sure if the m could be greater than 1.
    $endgroup$
    – Daniel Gimpelman
    Dec 15 '18 at 16:25










4




4




$begingroup$
What's wrong with $n=1+(n-1)$? Perhaps you meant to require $a,b>1$ but you should specify that.
$endgroup$
– lulu
Dec 15 '18 at 16:03






$begingroup$
What's wrong with $n=1+(n-1)$? Perhaps you meant to require $a,b>1$ but you should specify that.
$endgroup$
– lulu
Dec 15 '18 at 16:03






1




1




$begingroup$
Dirichlet’s theorem is useless in that kind of situation; the problem is much, much simpler. Aside from the obvious case $n=(n-1)+1$, you can try to find general patterns when $n=4m$, $n=2m+1$, $n=4m+2$.
$endgroup$
– Mindlack
Dec 15 '18 at 16:05




$begingroup$
Dirichlet’s theorem is useless in that kind of situation; the problem is much, much simpler. Aside from the obvious case $n=(n-1)+1$, you can try to find general patterns when $n=4m$, $n=2m+1$, $n=4m+2$.
$endgroup$
– Mindlack
Dec 15 '18 at 16:05




1




1




$begingroup$
True i should have specified that a,b>1, sorry, my bad.
$endgroup$
– Daniel Gimpelman
Dec 15 '18 at 16:07




$begingroup$
True i should have specified that a,b>1, sorry, my bad.
$endgroup$
– Daniel Gimpelman
Dec 15 '18 at 16:07












$begingroup$
Daniel, you can edit your post. Also, have a look at this introduction to posting mathematical notation.
$endgroup$
– hardmath
Dec 15 '18 at 16:12




$begingroup$
Daniel, you can edit your post. Also, have a look at this introduction to posting mathematical notation.
$endgroup$
– hardmath
Dec 15 '18 at 16:12












$begingroup$
Mindlack thanks, but I don't really understand how to go from here. How do I represent a and b even in a specific case? $7=2*3+1$ where $a=2$ and $b=1$? $8=4*2$ where $a=4$ and $b=0$? (And than b isn't co-prime). $10=4*2+2$ where $a=4$ and $b=2$ (a,b aren't co-prime). Also there would always be the m, which i'm not sure if the m could be greater than 1.
$endgroup$
– Daniel Gimpelman
Dec 15 '18 at 16:25






$begingroup$
Mindlack thanks, but I don't really understand how to go from here. How do I represent a and b even in a specific case? $7=2*3+1$ where $a=2$ and $b=1$? $8=4*2$ where $a=4$ and $b=0$? (And than b isn't co-prime). $10=4*2+2$ where $a=4$ and $b=2$ (a,b aren't co-prime). Also there would always be the m, which i'm not sure if the m could be greater than 1.
$endgroup$
– Daniel Gimpelman
Dec 15 '18 at 16:25












4 Answers
4






active

oldest

votes


















2












$begingroup$

Well if $n$ is odd you can always do $n-2$ and $2$. Or you can do $frac {n-1}2$ and $frac {n+1}2$.



If $n = 2k$ and $k$ is even you can do $k-1$ and $k+1$. As $kpm 1$ is odd and $gcd(k-1, k+1) = gcd(k-1, k+1 -(k-1)) = gcd(k-1,2)=1$.



If $n = 2k$ and $k$ is odd you can do $k-2$ and $k+2$ and as $kpm 2$ is odd you have $gcd(k-2,k+2)=gcd(k-1, 4) = 1$.






share|cite|improve this answer









$endgroup$





















    6












    $begingroup$

    Just to provide an answer synthesized out of the comments already posted, your best (read as easiest) approach to this kind of problem is to toy around with general patterns until something either clicks and you can write a clever proof or until you accidentally exhaust all possible cases.



    In this particular problem, we can break down cases into the residue classes $bmod 4$ in order to hunt for patterns:



    1) If $n=2k+1$ then the decomposition $n=(k)+(k+1)$ satisfies our criterion since consecutive numbers are always coprime and $kgeq 3$.



    2) If $n=4k$ then consider the decomposition $n=(2k-1)+(2k+1)$. Are these numbers coprime? We can no longer rely upon the general fact that consecutive numbers are coprime, since these are not consecutive. However, if two numbers differ by exactly $2$, what is the only prime factor that they can share? In general, if two numbers differ by $m$, what prime factors can they share? Finally, are we sure that these numbers are both greater than $1$?



    I have basically given away the entire answer, but I didn't know how to discuss this phenomenon in any other way, so I leave the final details of the second case, and the entirety of the third case, to you.






    share|cite|improve this answer









    $endgroup$









    • 2




      $begingroup$
      +1 A nice combination of hint and solution.
      $endgroup$
      – Ethan Bolker
      Dec 15 '18 at 16:25



















    1












    $begingroup$

    Later: the numbers between $1$ and $n-1$ that are relatively prime to $n$ itself come in pairs that add up to $n$ and are relatively prime to each other as well. If $n=5$ or $n geq 7$ both such numbers can be chosen strictly larger than $1.$



    Original:



    A different emphasis: if Euler's totient $phi(n) geq 3,$ then there is some integer $a$ with $gcd(a,n) = 1$ and $1 < a < n-1.$ If we then name $b = n-a,$ we find that $gcd(a,b) = 1$ as well, since a prime $p$ that divides both $a,n-a$ also divides $n,$ and this contradicts $gcd(a,n) = 1.$



    So, when is $phi(n) geq 3 ; ? ; ;$ If $n$ is divisible by any prime $q geq 5,$ then $phi(n)$ is a multiple of $phi(q) = q-1,$ and that is at least $4.$



    Next, if $n = 2^c ; 3^d ; . ;$ When $d=0$ we find $phi(n) = 2^{c-1}$ is at least $3$ when $c geq 3,$ leaving $2,4$ out. When $c=0$ we find $phi(n) = 2 cdot 3^{d-1}$ is at least $3$ when $d geq 2,$ leaving $3$ out. When $c,d geq 1,$ we find $phi(n) = 2^c cdot 3^{d-1}$ is at least $3$ when either $c geq 2$ or $d geq 2,$ so this leaves out $6.$



    Put it together, for $n=5$ or $n geq 7,$ there is some $a$ with $1 < a < n-1$ and $gcd(a,n) = 1.$






    share|cite|improve this answer











    $endgroup$





















      1












      $begingroup$

      Here's another route you can take to solve this problem. For any $n ge 7$, you want to show that there is a number $a$ where





      1. $gcd(a, n - a) = 1$,


      2. $1 < a < n$, and


      3. $1 < n - a < n$.


      One option would be to choose $a$ to be the smallest prime number that doesn't divide $n$. In that case, $gcd(a, n - a) = 1$ because otherwise you'd have $gcd(a, n - a) = a$, meaning that $a$ divides $a + (n - a) = n$, contradicting the fact that $a$ doesn't divide $n$.



      What you'll need to then show is that if you pick $n ge 7$ that the smallest prime number that doesn't divide $n$ happens to be less than $n - 1$. I'll leave that as an exercise to the reader. :-)






      share|cite|improve this answer









      $endgroup$













        Your Answer





        StackExchange.ifUsing("editor", function () {
        return StackExchange.using("mathjaxEditing", function () {
        StackExchange.MarkdownEditor.creationCallbacks.add(function (editor, postfix) {
        StackExchange.mathjaxEditing.prepareWmdForMathJax(editor, postfix, [["$", "$"], ["\\(","\\)"]]);
        });
        });
        }, "mathjax-editing");

        StackExchange.ready(function() {
        var channelOptions = {
        tags: "".split(" "),
        id: "69"
        };
        initTagRenderer("".split(" "), "".split(" "), channelOptions);

        StackExchange.using("externalEditor", function() {
        // Have to fire editor after snippets, if snippets enabled
        if (StackExchange.settings.snippets.snippetsEnabled) {
        StackExchange.using("snippets", function() {
        createEditor();
        });
        }
        else {
        createEditor();
        }
        });

        function createEditor() {
        StackExchange.prepareEditor({
        heartbeatType: 'answer',
        autoActivateHeartbeat: false,
        convertImagesToLinks: true,
        noModals: true,
        showLowRepImageUploadWarning: true,
        reputationToPostImages: 10,
        bindNavPrevention: true,
        postfix: "",
        imageUploader: {
        brandingHtml: "Powered by u003ca class="icon-imgur-white" href="https://imgur.com/"u003eu003c/au003e",
        contentPolicyHtml: "User contributions licensed under u003ca href="https://creativecommons.org/licenses/by-sa/3.0/"u003ecc by-sa 3.0 with attribution requiredu003c/au003e u003ca href="https://stackoverflow.com/legal/content-policy"u003e(content policy)u003c/au003e",
        allowUrls: true
        },
        noCode: true, onDemand: true,
        discardSelector: ".discard-answer"
        ,immediatelyShowMarkdownHelp:true
        });


        }
        });














        draft saved

        draft discarded


















        StackExchange.ready(
        function () {
        StackExchange.openid.initPostLogin('.new-post-login', 'https%3a%2f%2fmath.stackexchange.com%2fquestions%2f3041656%2fsum-of-two-co-prime-integers%23new-answer', 'question_page');
        }
        );

        Post as a guest















        Required, but never shown

























        4 Answers
        4






        active

        oldest

        votes








        4 Answers
        4






        active

        oldest

        votes









        active

        oldest

        votes






        active

        oldest

        votes









        2












        $begingroup$

        Well if $n$ is odd you can always do $n-2$ and $2$. Or you can do $frac {n-1}2$ and $frac {n+1}2$.



        If $n = 2k$ and $k$ is even you can do $k-1$ and $k+1$. As $kpm 1$ is odd and $gcd(k-1, k+1) = gcd(k-1, k+1 -(k-1)) = gcd(k-1,2)=1$.



        If $n = 2k$ and $k$ is odd you can do $k-2$ and $k+2$ and as $kpm 2$ is odd you have $gcd(k-2,k+2)=gcd(k-1, 4) = 1$.






        share|cite|improve this answer









        $endgroup$


















          2












          $begingroup$

          Well if $n$ is odd you can always do $n-2$ and $2$. Or you can do $frac {n-1}2$ and $frac {n+1}2$.



          If $n = 2k$ and $k$ is even you can do $k-1$ and $k+1$. As $kpm 1$ is odd and $gcd(k-1, k+1) = gcd(k-1, k+1 -(k-1)) = gcd(k-1,2)=1$.



          If $n = 2k$ and $k$ is odd you can do $k-2$ and $k+2$ and as $kpm 2$ is odd you have $gcd(k-2,k+2)=gcd(k-1, 4) = 1$.






          share|cite|improve this answer









          $endgroup$
















            2












            2








            2





            $begingroup$

            Well if $n$ is odd you can always do $n-2$ and $2$. Or you can do $frac {n-1}2$ and $frac {n+1}2$.



            If $n = 2k$ and $k$ is even you can do $k-1$ and $k+1$. As $kpm 1$ is odd and $gcd(k-1, k+1) = gcd(k-1, k+1 -(k-1)) = gcd(k-1,2)=1$.



            If $n = 2k$ and $k$ is odd you can do $k-2$ and $k+2$ and as $kpm 2$ is odd you have $gcd(k-2,k+2)=gcd(k-1, 4) = 1$.






            share|cite|improve this answer









            $endgroup$



            Well if $n$ is odd you can always do $n-2$ and $2$. Or you can do $frac {n-1}2$ and $frac {n+1}2$.



            If $n = 2k$ and $k$ is even you can do $k-1$ and $k+1$. As $kpm 1$ is odd and $gcd(k-1, k+1) = gcd(k-1, k+1 -(k-1)) = gcd(k-1,2)=1$.



            If $n = 2k$ and $k$ is odd you can do $k-2$ and $k+2$ and as $kpm 2$ is odd you have $gcd(k-2,k+2)=gcd(k-1, 4) = 1$.







            share|cite|improve this answer












            share|cite|improve this answer



            share|cite|improve this answer










            answered Dec 15 '18 at 21:35









            fleabloodfleablood

            68.8k22685




            68.8k22685























                6












                $begingroup$

                Just to provide an answer synthesized out of the comments already posted, your best (read as easiest) approach to this kind of problem is to toy around with general patterns until something either clicks and you can write a clever proof or until you accidentally exhaust all possible cases.



                In this particular problem, we can break down cases into the residue classes $bmod 4$ in order to hunt for patterns:



                1) If $n=2k+1$ then the decomposition $n=(k)+(k+1)$ satisfies our criterion since consecutive numbers are always coprime and $kgeq 3$.



                2) If $n=4k$ then consider the decomposition $n=(2k-1)+(2k+1)$. Are these numbers coprime? We can no longer rely upon the general fact that consecutive numbers are coprime, since these are not consecutive. However, if two numbers differ by exactly $2$, what is the only prime factor that they can share? In general, if two numbers differ by $m$, what prime factors can they share? Finally, are we sure that these numbers are both greater than $1$?



                I have basically given away the entire answer, but I didn't know how to discuss this phenomenon in any other way, so I leave the final details of the second case, and the entirety of the third case, to you.






                share|cite|improve this answer









                $endgroup$









                • 2




                  $begingroup$
                  +1 A nice combination of hint and solution.
                  $endgroup$
                  – Ethan Bolker
                  Dec 15 '18 at 16:25
















                6












                $begingroup$

                Just to provide an answer synthesized out of the comments already posted, your best (read as easiest) approach to this kind of problem is to toy around with general patterns until something either clicks and you can write a clever proof or until you accidentally exhaust all possible cases.



                In this particular problem, we can break down cases into the residue classes $bmod 4$ in order to hunt for patterns:



                1) If $n=2k+1$ then the decomposition $n=(k)+(k+1)$ satisfies our criterion since consecutive numbers are always coprime and $kgeq 3$.



                2) If $n=4k$ then consider the decomposition $n=(2k-1)+(2k+1)$. Are these numbers coprime? We can no longer rely upon the general fact that consecutive numbers are coprime, since these are not consecutive. However, if two numbers differ by exactly $2$, what is the only prime factor that they can share? In general, if two numbers differ by $m$, what prime factors can they share? Finally, are we sure that these numbers are both greater than $1$?



                I have basically given away the entire answer, but I didn't know how to discuss this phenomenon in any other way, so I leave the final details of the second case, and the entirety of the third case, to you.






                share|cite|improve this answer









                $endgroup$









                • 2




                  $begingroup$
                  +1 A nice combination of hint and solution.
                  $endgroup$
                  – Ethan Bolker
                  Dec 15 '18 at 16:25














                6












                6








                6





                $begingroup$

                Just to provide an answer synthesized out of the comments already posted, your best (read as easiest) approach to this kind of problem is to toy around with general patterns until something either clicks and you can write a clever proof or until you accidentally exhaust all possible cases.



                In this particular problem, we can break down cases into the residue classes $bmod 4$ in order to hunt for patterns:



                1) If $n=2k+1$ then the decomposition $n=(k)+(k+1)$ satisfies our criterion since consecutive numbers are always coprime and $kgeq 3$.



                2) If $n=4k$ then consider the decomposition $n=(2k-1)+(2k+1)$. Are these numbers coprime? We can no longer rely upon the general fact that consecutive numbers are coprime, since these are not consecutive. However, if two numbers differ by exactly $2$, what is the only prime factor that they can share? In general, if two numbers differ by $m$, what prime factors can they share? Finally, are we sure that these numbers are both greater than $1$?



                I have basically given away the entire answer, but I didn't know how to discuss this phenomenon in any other way, so I leave the final details of the second case, and the entirety of the third case, to you.






                share|cite|improve this answer









                $endgroup$



                Just to provide an answer synthesized out of the comments already posted, your best (read as easiest) approach to this kind of problem is to toy around with general patterns until something either clicks and you can write a clever proof or until you accidentally exhaust all possible cases.



                In this particular problem, we can break down cases into the residue classes $bmod 4$ in order to hunt for patterns:



                1) If $n=2k+1$ then the decomposition $n=(k)+(k+1)$ satisfies our criterion since consecutive numbers are always coprime and $kgeq 3$.



                2) If $n=4k$ then consider the decomposition $n=(2k-1)+(2k+1)$. Are these numbers coprime? We can no longer rely upon the general fact that consecutive numbers are coprime, since these are not consecutive. However, if two numbers differ by exactly $2$, what is the only prime factor that they can share? In general, if two numbers differ by $m$, what prime factors can they share? Finally, are we sure that these numbers are both greater than $1$?



                I have basically given away the entire answer, but I didn't know how to discuss this phenomenon in any other way, so I leave the final details of the second case, and the entirety of the third case, to you.







                share|cite|improve this answer












                share|cite|improve this answer



                share|cite|improve this answer










                answered Dec 15 '18 at 16:23









                RandomMathDudeRandomMathDude

                1763




                1763








                • 2




                  $begingroup$
                  +1 A nice combination of hint and solution.
                  $endgroup$
                  – Ethan Bolker
                  Dec 15 '18 at 16:25














                • 2




                  $begingroup$
                  +1 A nice combination of hint and solution.
                  $endgroup$
                  – Ethan Bolker
                  Dec 15 '18 at 16:25








                2




                2




                $begingroup$
                +1 A nice combination of hint and solution.
                $endgroup$
                – Ethan Bolker
                Dec 15 '18 at 16:25




                $begingroup$
                +1 A nice combination of hint and solution.
                $endgroup$
                – Ethan Bolker
                Dec 15 '18 at 16:25











                1












                $begingroup$

                Later: the numbers between $1$ and $n-1$ that are relatively prime to $n$ itself come in pairs that add up to $n$ and are relatively prime to each other as well. If $n=5$ or $n geq 7$ both such numbers can be chosen strictly larger than $1.$



                Original:



                A different emphasis: if Euler's totient $phi(n) geq 3,$ then there is some integer $a$ with $gcd(a,n) = 1$ and $1 < a < n-1.$ If we then name $b = n-a,$ we find that $gcd(a,b) = 1$ as well, since a prime $p$ that divides both $a,n-a$ also divides $n,$ and this contradicts $gcd(a,n) = 1.$



                So, when is $phi(n) geq 3 ; ? ; ;$ If $n$ is divisible by any prime $q geq 5,$ then $phi(n)$ is a multiple of $phi(q) = q-1,$ and that is at least $4.$



                Next, if $n = 2^c ; 3^d ; . ;$ When $d=0$ we find $phi(n) = 2^{c-1}$ is at least $3$ when $c geq 3,$ leaving $2,4$ out. When $c=0$ we find $phi(n) = 2 cdot 3^{d-1}$ is at least $3$ when $d geq 2,$ leaving $3$ out. When $c,d geq 1,$ we find $phi(n) = 2^c cdot 3^{d-1}$ is at least $3$ when either $c geq 2$ or $d geq 2,$ so this leaves out $6.$



                Put it together, for $n=5$ or $n geq 7,$ there is some $a$ with $1 < a < n-1$ and $gcd(a,n) = 1.$






                share|cite|improve this answer











                $endgroup$


















                  1












                  $begingroup$

                  Later: the numbers between $1$ and $n-1$ that are relatively prime to $n$ itself come in pairs that add up to $n$ and are relatively prime to each other as well. If $n=5$ or $n geq 7$ both such numbers can be chosen strictly larger than $1.$



                  Original:



                  A different emphasis: if Euler's totient $phi(n) geq 3,$ then there is some integer $a$ with $gcd(a,n) = 1$ and $1 < a < n-1.$ If we then name $b = n-a,$ we find that $gcd(a,b) = 1$ as well, since a prime $p$ that divides both $a,n-a$ also divides $n,$ and this contradicts $gcd(a,n) = 1.$



                  So, when is $phi(n) geq 3 ; ? ; ;$ If $n$ is divisible by any prime $q geq 5,$ then $phi(n)$ is a multiple of $phi(q) = q-1,$ and that is at least $4.$



                  Next, if $n = 2^c ; 3^d ; . ;$ When $d=0$ we find $phi(n) = 2^{c-1}$ is at least $3$ when $c geq 3,$ leaving $2,4$ out. When $c=0$ we find $phi(n) = 2 cdot 3^{d-1}$ is at least $3$ when $d geq 2,$ leaving $3$ out. When $c,d geq 1,$ we find $phi(n) = 2^c cdot 3^{d-1}$ is at least $3$ when either $c geq 2$ or $d geq 2,$ so this leaves out $6.$



                  Put it together, for $n=5$ or $n geq 7,$ there is some $a$ with $1 < a < n-1$ and $gcd(a,n) = 1.$






                  share|cite|improve this answer











                  $endgroup$
















                    1












                    1








                    1





                    $begingroup$

                    Later: the numbers between $1$ and $n-1$ that are relatively prime to $n$ itself come in pairs that add up to $n$ and are relatively prime to each other as well. If $n=5$ or $n geq 7$ both such numbers can be chosen strictly larger than $1.$



                    Original:



                    A different emphasis: if Euler's totient $phi(n) geq 3,$ then there is some integer $a$ with $gcd(a,n) = 1$ and $1 < a < n-1.$ If we then name $b = n-a,$ we find that $gcd(a,b) = 1$ as well, since a prime $p$ that divides both $a,n-a$ also divides $n,$ and this contradicts $gcd(a,n) = 1.$



                    So, when is $phi(n) geq 3 ; ? ; ;$ If $n$ is divisible by any prime $q geq 5,$ then $phi(n)$ is a multiple of $phi(q) = q-1,$ and that is at least $4.$



                    Next, if $n = 2^c ; 3^d ; . ;$ When $d=0$ we find $phi(n) = 2^{c-1}$ is at least $3$ when $c geq 3,$ leaving $2,4$ out. When $c=0$ we find $phi(n) = 2 cdot 3^{d-1}$ is at least $3$ when $d geq 2,$ leaving $3$ out. When $c,d geq 1,$ we find $phi(n) = 2^c cdot 3^{d-1}$ is at least $3$ when either $c geq 2$ or $d geq 2,$ so this leaves out $6.$



                    Put it together, for $n=5$ or $n geq 7,$ there is some $a$ with $1 < a < n-1$ and $gcd(a,n) = 1.$






                    share|cite|improve this answer











                    $endgroup$



                    Later: the numbers between $1$ and $n-1$ that are relatively prime to $n$ itself come in pairs that add up to $n$ and are relatively prime to each other as well. If $n=5$ or $n geq 7$ both such numbers can be chosen strictly larger than $1.$



                    Original:



                    A different emphasis: if Euler's totient $phi(n) geq 3,$ then there is some integer $a$ with $gcd(a,n) = 1$ and $1 < a < n-1.$ If we then name $b = n-a,$ we find that $gcd(a,b) = 1$ as well, since a prime $p$ that divides both $a,n-a$ also divides $n,$ and this contradicts $gcd(a,n) = 1.$



                    So, when is $phi(n) geq 3 ; ? ; ;$ If $n$ is divisible by any prime $q geq 5,$ then $phi(n)$ is a multiple of $phi(q) = q-1,$ and that is at least $4.$



                    Next, if $n = 2^c ; 3^d ; . ;$ When $d=0$ we find $phi(n) = 2^{c-1}$ is at least $3$ when $c geq 3,$ leaving $2,4$ out. When $c=0$ we find $phi(n) = 2 cdot 3^{d-1}$ is at least $3$ when $d geq 2,$ leaving $3$ out. When $c,d geq 1,$ we find $phi(n) = 2^c cdot 3^{d-1}$ is at least $3$ when either $c geq 2$ or $d geq 2,$ so this leaves out $6.$



                    Put it together, for $n=5$ or $n geq 7,$ there is some $a$ with $1 < a < n-1$ and $gcd(a,n) = 1.$







                    share|cite|improve this answer














                    share|cite|improve this answer



                    share|cite|improve this answer








                    edited Dec 15 '18 at 21:21

























                    answered Dec 15 '18 at 18:33









                    Will JagyWill Jagy

                    102k5101199




                    102k5101199























                        1












                        $begingroup$

                        Here's another route you can take to solve this problem. For any $n ge 7$, you want to show that there is a number $a$ where





                        1. $gcd(a, n - a) = 1$,


                        2. $1 < a < n$, and


                        3. $1 < n - a < n$.


                        One option would be to choose $a$ to be the smallest prime number that doesn't divide $n$. In that case, $gcd(a, n - a) = 1$ because otherwise you'd have $gcd(a, n - a) = a$, meaning that $a$ divides $a + (n - a) = n$, contradicting the fact that $a$ doesn't divide $n$.



                        What you'll need to then show is that if you pick $n ge 7$ that the smallest prime number that doesn't divide $n$ happens to be less than $n - 1$. I'll leave that as an exercise to the reader. :-)






                        share|cite|improve this answer









                        $endgroup$


















                          1












                          $begingroup$

                          Here's another route you can take to solve this problem. For any $n ge 7$, you want to show that there is a number $a$ where





                          1. $gcd(a, n - a) = 1$,


                          2. $1 < a < n$, and


                          3. $1 < n - a < n$.


                          One option would be to choose $a$ to be the smallest prime number that doesn't divide $n$. In that case, $gcd(a, n - a) = 1$ because otherwise you'd have $gcd(a, n - a) = a$, meaning that $a$ divides $a + (n - a) = n$, contradicting the fact that $a$ doesn't divide $n$.



                          What you'll need to then show is that if you pick $n ge 7$ that the smallest prime number that doesn't divide $n$ happens to be less than $n - 1$. I'll leave that as an exercise to the reader. :-)






                          share|cite|improve this answer









                          $endgroup$
















                            1












                            1








                            1





                            $begingroup$

                            Here's another route you can take to solve this problem. For any $n ge 7$, you want to show that there is a number $a$ where





                            1. $gcd(a, n - a) = 1$,


                            2. $1 < a < n$, and


                            3. $1 < n - a < n$.


                            One option would be to choose $a$ to be the smallest prime number that doesn't divide $n$. In that case, $gcd(a, n - a) = 1$ because otherwise you'd have $gcd(a, n - a) = a$, meaning that $a$ divides $a + (n - a) = n$, contradicting the fact that $a$ doesn't divide $n$.



                            What you'll need to then show is that if you pick $n ge 7$ that the smallest prime number that doesn't divide $n$ happens to be less than $n - 1$. I'll leave that as an exercise to the reader. :-)






                            share|cite|improve this answer









                            $endgroup$



                            Here's another route you can take to solve this problem. For any $n ge 7$, you want to show that there is a number $a$ where





                            1. $gcd(a, n - a) = 1$,


                            2. $1 < a < n$, and


                            3. $1 < n - a < n$.


                            One option would be to choose $a$ to be the smallest prime number that doesn't divide $n$. In that case, $gcd(a, n - a) = 1$ because otherwise you'd have $gcd(a, n - a) = a$, meaning that $a$ divides $a + (n - a) = n$, contradicting the fact that $a$ doesn't divide $n$.



                            What you'll need to then show is that if you pick $n ge 7$ that the smallest prime number that doesn't divide $n$ happens to be less than $n - 1$. I'll leave that as an exercise to the reader. :-)







                            share|cite|improve this answer












                            share|cite|improve this answer



                            share|cite|improve this answer










                            answered Dec 15 '18 at 22:46









                            templatetypedeftemplatetypedef

                            4,56622457




                            4,56622457






























                                draft saved

                                draft discarded




















































                                Thanks for contributing an answer to Mathematics Stack Exchange!


                                • Please be sure to answer the question. Provide details and share your research!

                                But avoid



                                • Asking for help, clarification, or responding to other answers.

                                • Making statements based on opinion; back them up with references or personal experience.


                                Use MathJax to format equations. MathJax reference.


                                To learn more, see our tips on writing great answers.




                                draft saved


                                draft discarded














                                StackExchange.ready(
                                function () {
                                StackExchange.openid.initPostLogin('.new-post-login', 'https%3a%2f%2fmath.stackexchange.com%2fquestions%2f3041656%2fsum-of-two-co-prime-integers%23new-answer', 'question_page');
                                }
                                );

                                Post as a guest















                                Required, but never shown





















































                                Required, but never shown














                                Required, but never shown












                                Required, but never shown







                                Required, but never shown

































                                Required, but never shown














                                Required, but never shown












                                Required, but never shown







                                Required, but never shown







                                Popular posts from this blog

                                Список кардиналов, возведённых папой римским Каликстом III

                                Deduzione

                                Mysql.sock missing - “Can't connect to local MySQL server through socket”